Difference between revisions of "1993 IMO Problems/Problem 1"
(→Alternate Solution) |
|||
Line 14: | Line 14: | ||
Q.E.D. | Q.E.D. | ||
+ | |||
+ | == Solution 2 == | ||
+ | Let us assume the contrary. If <math>f(x)=p(x)q(x)</math>, then that is equivalent to <math>x^{n-1}(x+5)=p(x)q(x)-3</math>. Now we see that <math>p(0)q(0)=3</math> and <math>p(-5)q(-5)=3</math>. This implies that the constant terms of <math>p</math> and <math>q</math> are either <math>-1,-3</math> or <math>1,3</math>, up to permutation. Let <math>p(x)=a(x)+3</math> and <math>q(x)=b(x)+1</math>. The only possibility for <math>p(-5)q(-5)</math> to be equal to <math>3</math> is to make <math>a(-5)=b(-5)=0</math>. This is because if they are not zero they must be nonzero multiples of 5, and so <math>p(-5)q(-5)</math> must be larger than 3. So <math>p(x)-3=(x+5)A(x)</math> and <math>q(x)-1=(x+5)B(x)</math>. Now plug these back to <math>x^{n-1}(x+5)=p(x)q(x)-3</math> we see that <cmath>x^{n-1}(x+5)=[(x+5)A(x)+3][(x+5)B(x)+1]-3</cmath>. This is equivalent to <cmath>(x+5)^2A(x)B(x)+(x+5)A(x)+3(x+5)B(x)=(x+5)x^{n-1}</cmath>. One of <math>A(x)</math> and <math>B(x)</math> must be 0. But then that implies one of <math>p</math> and <math>q</math> is less than degree 1. So the assumption must be false. The <math>(-1,-3)</math> case is similar. | ||
+ | |||
==Alternate Solution== | ==Alternate Solution== |
Revision as of 22:23, 28 September 2025
Problem
Let , where
is an integer. Prove that
cannot be expressed as the product of two non-constant polynomials with integer coefficients.
Solution
For the sake of contradiction, assume that for polynomials
and
in
. Furthermore, let
with
if
and
with
if
. This gives that
.
We have that , or
. WLOG, let
(and thus
). Since
and
divides
but not
, we need that
. We can keep on going up the chain until we get that
. Then, by equating coefficients once more, we get that
. Taking the equation
gives that
. This implies that
. Thus, the degree of
is at least
. However, because
is a non-constant factor of
, we have that the degree of
is at most
. Thus, the degree of
is
, implying that the degree of
is
.
From this fact, we have that there must exist an integer root of . However, when
is an integer,
, meaning
is odd, so
cannot be a root of
.
Hence, cannot be expressed as
for polynomials
and
in
. This means that
cannot be expressed as the product of two non-constant polynomials with integer coefficients.
Q.E.D.
Solution 2
Let us assume the contrary. If , then that is equivalent to
. Now we see that
and
. This implies that the constant terms of
and
are either
or
, up to permutation. Let
and
. The only possibility for
to be equal to
is to make
. This is because if they are not zero they must be nonzero multiples of 5, and so
must be larger than 3. So
and
. Now plug these back to
we see that
. This is equivalent to
. One of
and
must be 0. But then that implies one of
and
is less than degree 1. So the assumption must be false. The
case is similar.
Alternate Solution
It’s actually trivial by Perron’s Criterion lol
Note: Quoting Perron's Criterion on the actual IMO will very likely result in a score in the set , since it was not a well-known result back then.
See Also
1993 IMO (Problems) • Resources | ||
Preceded by First Question |
1 • 2 • 3 • 4 • 5 • 6 | Followed by Problem 2 |
All IMO Problems and Solutions |